Proving a Claim in Ticcati's Red QFT Textbook: Seeking Advice and Hints

  • Thread starter MathematicalPhysicist
  • Start date
In summary: The non-rigorous argument for Y^r-X^r is that when you subtract two components of X from each other, the result is going to be invariant under rotations.
  • #1
MathematicalPhysicist
Gold Member
4,699
373
Does anybody know how prove the claim in my question here:
http://theoreticalphysics.stackexchange.com/questions/643/a-question-from-ticcatis-red-qft-textbook

thanks, any hints?
 
Last edited by a moderator:
Physics news on Phys.org
  • #2
I have another question from the same textbook.

In Remark 1.6.4 he starts to show that the position operator is unique, so we take two such operators [itex]X,Y[/itex], and he assumes that [itex]Y[/itex] is given wrt the basis [itex]|k>[/itex]. The canonical commutation must be satisified for both of them, then [itex]P_r[/itex] commutes with [itex]X_s-Y_s[/itex] (which thusfar I follow). Therefore assuming that any operator can be expressed as a function of both [itex]X_r,P_r[/itex], [itex]Y_r[/itex] must be of the form [itex]X_r+f_r(P)[/itex], now the thing that I don't understand is that he argues Axiom 3 implies that [itex]f_r(P)[/itex] is of the form [itex]g(|P|^2)P_r[/itex], where axiom 3 tells us that if [itex]R[/itex] is a space rotation, then [itex]U(R)^{\dagger} X U(R) = RX[/itex] where [itex]U(R)[/itex] is the unitary representation of [itex]R[/itex] in Poincare group.

Any help as to why Ax 3 implies it?
I don't see it.
 
  • #3
MathematicalPhysicist said:
Does anybody know how prove the claim in my question here:
http://theoreticalphysics.stackexchange.com/questions/643/a-question-from-ticcatis-red-qft-textbook

thanks, any hints?
That guy Pavel solved it for you.

MathematicalPhysicist said:
he assumes that [itex]Y[/itex] is given wrt the basis [itex]|k>[/itex].
I don't understand what this means. I understand that |k> is a momentum eigenket, but that doesn't help. The exact statement in the book is "Assume that [itex]\bar Y[/itex] is the position operator with respect to the basis [itex]|\bar k\rangle[/itex]". I still don't get it. There's no basis involved in the axioms for the position operator.


MathematicalPhysicist said:
...now the thing that I don't understand is that he argues Axiom 3 implies that [itex]f_r(P)[/itex] is of the form [itex]g(|P|^2)P_r[/itex], where axiom 3 tells us that if [itex]R[/itex] is a space rotation, then [itex]U(R)^{\dagger} X U(R) = RX[/itex] where [itex]U(R)[/itex] is the unitary representation of [itex]R[/itex] in Poincare group.
The equality in axiom 3 is [itex]U(R)^\dagger\bar X U(R)=R\bar X[/itex]. This has to mean [itex]U(R)^\dagger X^r U(R)=R^r_s X^s[/itex]. So [itex]Y^r=X^r+f^r(\bar P)[/itex] implies [itex]R^r_sY^s=R^r_sX^s+U(R)^\dagger f^r(\bar P)U(R)[/itex]. Multiply this by [itex](R^{-1})^t_r[/itex], and the result is [itex]Y^t=X^t+U(R)^\dagger(R^{-1})^t_r f^r(\bar P) U(R)[/itex]. So [tex] f^r(\bar P)=Y^r-X^r=U(R)^\dagger(R^{-1})^r_s f^s(\bar P) U(R).[/tex] Does this help, or is this the point where you are stuck?
 
Last edited by a moderator:
  • #4
Do you assume that [itex]R^{-1}[/itex] commutes with [itex]U(R)^{\dagger}[/itex]?

Assuming you do, I still don't see how from your last line you get that [itex]f_r(P)[/itex] equals [itex]g(|P|^2) P_r[/itex].

Edit: I guess it's reasonable to argue that R and its inverse will commute with their Poincare representations, I think.
 
Last edited:
  • #5
There's no need to assume it. [itex]R^r_s[/itex] is the number on row r, column s, of the rotation matrix R. When it appears as a factor in a formula involving operators, it should be interpreted as the real number [itex]R^r_s[/itex] times the identity operator, and the identity operator commutes with everything.

I assume that you can see that the converse of what you want to prove is true, i.e. that if [itex]f^r(\bar P)[/itex] is what you need it to be, then the last equality of my previous post holds. It's harder to explain why [itex]f^r(\bar P)[/itex] must be of that form. I haven't worked it out, but I think it's essential that the last equality in my previous post is supposed to hold for all R. Maybe it will help to consider a few specific choices of R.
 
Last edited:
  • #6
I think I understand why if [itex]f^r(\bar{P})=g(|P|^2)P^r[/itex] then it satisfies the relation you wrote with the unitary representations, it's because the 4-momentum vector is invariant under space rotations.

Not sure about the converse.

P.S
Appreciate your help. :-)
 
  • #7
[itex]\bar P[/itex] is the 3-momentum vector. Its components aren't invariant under rotation, so the components of the 4-momentum vector aren't either. But [itex]|\bar P|^2[/itex] is invariant, because a rotation by definition doesn't change the norm of any vector. Edit: To be more precise, in a notation that puts all indices downstairs (because the "one upstairs, one downstairs" notation makes it hard to handle transposes and inverses), [tex]U(R)^\dagger\bar P^2U(R)=U(R)^\dagger P_r U(R)\ U(R)^\dagger P_r U(R)=R_{rs}P_s R_{rt} P_t=(R^T)_{sr}R_{rt}P_sP_t=\delta_{st}P_sP_t=P_sP_s=\bar P^2.[/tex] I don't think there's an easy way to prove that [itex]f^r(\bar P)[/itex] must be of the form [itex]g(|\bar P|^2)P^r[/itex]. I also don't think there's an easy way to prove that [itex]Y^r-X^r[/itex] must be of the form [itex]f^r(\bar P)[/itex]. Ticciati doesn't even explain what that means. I'm going to think about this some more, but I can't guarantee that I will come up with something useful.

The non-rigorous argument for the form of [itex]f^r(\bar P)[/itex] is that when you multiply components of [itex]\bar P[/itex] together and sum over repeated indices, the result isn't going to transform under rotations like a 3-vector unless r is the only index left. Terms like [itex]P^rP^s P_s P^t P_t[/itex] are OK, but terms like [itex]P^r P^s[/itex] aren't.
 
Last edited:

FAQ: Proving a Claim in Ticcati's Red QFT Textbook: Seeking Advice and Hints

What is Ticcati's Red QFT Textbook?

Ticcati's Red QFT Textbook is a theoretical physics textbook that presents a mathematical framework for quantum field theory.

What is the purpose of proving a claim in Ticcati's Red QFT Textbook?

The purpose of proving a claim in Ticcati's Red QFT Textbook is to demonstrate the validity of a statement or theorem using mathematical and logical reasoning.

Why is it important to seek advice and hints when proving a claim in Ticcati's Red QFT Textbook?

Seeking advice and hints from other scientists can provide valuable insights and perspectives that can help improve the quality and accuracy of the proof.

What are some tips for successfully proving a claim in Ticcati's Red QFT Textbook?

Some tips for successfully proving a claim in Ticcati's Red QFT Textbook include carefully reviewing the textbook and related literature, breaking down the problem into smaller parts, and seeking help and feedback from others.

What are some common challenges when proving a claim in Ticcati's Red QFT Textbook?

Some common challenges when proving a claim in Ticcati's Red QFT Textbook include understanding and applying complex mathematical concepts, finding the most efficient approach to the proof, and avoiding errors and oversights.

Similar threads

Replies
3
Views
1K
Replies
34
Views
4K
Replies
3
Views
2K
Replies
5
Views
1K
Replies
19
Views
2K
Replies
69
Views
5K
Replies
10
Views
1K
Back
Top